2016 AMC 10A Problems/Problem 14

Revision as of 18:59, 3 February 2016 by Warrenwangtennis (talk | contribs) (Solution)

Solution

The amount of twos in our sum ranges from $0$ to $1008$, with differences of $3$ because $2 \cdot 3 = lcm(2, 3)$.

The possible amount of twos is $\frac{1008 - 0}{2} + 1 \Rightarrow \boxed{\textbf{(C)} 337}$